This is a PSA question.

The stimulus says the builders of the Glen Veil apartment complex will not complete the complex unless a road connecting it to the town of Sierra is built. “Unless” is group 3 which means negate sufficient, so for the builders to complete the apartment complex, it is necessary that the connecting road be built (complete complex → road built). This is the first claim, and it is a conditional claim.

The next claim is a causation claim, not a conditional claim. It says the completed apartment complex would strengthen Sierra’s economy, and a stronger economy would benefit every Sierra resident. So I would write:

complete complex —cause→ strong economy —cause→ every resident benefits

And the prescriptive conclusion is that the residents of Sierra should vote to fund the construction of the proposed road.

Before we get into the answer choices, can you state in very general terms the descriptive-premise (P) → prescriptive-conclusion (C) we are looking for? It is something like if a construction project is necessary to another construction project whose consequence is a strengthened economy or some benefit to everyone, then residents should vote in favor of the first construction project. Obviously, it does not have to be this exact wording, but you are looking for some variant of this P → C. But watch out for C → P.

Which is the recurring trap that shows up in Answer Choice (B). It says the residents of a town should not vote in favor of a local tax to fund the construction project unless that construction project will produce results that benefit all of those residents. “Unless” is negate sufficient. If residents should vote to fund the construction project, then every resident benefits. You can bet money all day long that they will give you the right content but in the wrong logical order in PSA questions.

(B) should say if the construction project will produce results that benefit all residents, then the residents of the town should vote for it. The construction of the road is necessary for the construction of the apartment which benefits every resident, so you can say that the construction of the road will also produce results, albeit indirectly. This is not logically airtight, but you will see later on in (E) why this could work in the context of a PSA question.

Answer Choice (A) is also wrong, but for a different reason. (A) says if a construction project is necessary in order to strengthen a town’s economy, then the residents of the town should vote in favor of a local tax to fund that construction project.

While (A) is capable of delivering the desired conclusion, the premise does not trigger (A)'s sufficient conditions. We were never told of any necessary conditions to strengthen a town's economy. Building the road is necessary for the apartment complex which in turn would strengthen the economy, but that doesn't mean building the road is necessary for strengthening the economy. There may well be plenty of other ways to strengthen the economy without having to build that road.

One way you could fix (A) is by fixing the stimulus. The stimulus could say that in order for our economy to be strengthened, it is required that the apartment complex be completed. You could also fix (A) itself. If a construction project is necessary for another construction project whose consequence is that of a strengthened economy, then the residents should vote to fund it. This would trigger our stimulus.

See if you can come up with an analogy. Here's one. For me, wearing running shoes is necessary in order to go for a long run. Going for a long run results in me feeling better. (A) wants to claim that wearing running shoes is necessary for me to feel better.

Correct Answer Choice (E) says anyone who would benefit from the results of a construction project should vote in favor of a local tax to fund that project. First of all, (E) has the premise and the conclusion in the right location. If you would benefit, then you should vote in favor. This is the P → C.

You might object to (E) by pointing out that the construction project referred to in (E) could be the construction of the apartment complex. Yes, but there is nothing in (E) that forces only this interpretation. We could have just correctly identified the construction project as the building of the road.

You might also point out that building a road is only a necessary condition for the apartment complex, and the benefits to every resident are consequences of the apartment complex, not the road. You might say that we do not know the benefits of the road since it is just a necessary condition.

But this is not a Sufficient Assumption question. This is a PSA question. In context, the result of building the road cannot be nothing. The stimulus said the apartment complex will not be completed unless the road is built, so it seems reasonable to say that the result of the road being built is that the apartment gets completed, which has its attendant benefits, some of which can be attributed to the road.

I admit this is not logically airtight, but we are in PSA territory and we're dealing with causal logic. It is much more reasonable to make this argument as opposed to one required for (A), where we would have to say that just because the apartment complex would result in a strengthened economy, there is no other way to strengthen the economy. Meanwhile, (E) is only asking us to say that the consequences of the apartment complex can accrue to the road as well, given the relationship between the two projects.

Answer Choice (C) says whenever a town funds a construction project, it should do so by means of a tax rather than a debt. Nobody is contemplating a debt. If I was trying to argue that a town should fund its projects with tax and not with other sources, then maybe (C) would contribute. But as it stands, (C) is totally useless.

Answer Choice (D) says only those residents who will benefit from the results of a construction project should be required to pay a tax. The rule here sets us up for contraposing in order to yield the result that certain residents shouldn't be required to pay a tax. That's not what we want. We are only trying to argue that residents should vote in favor of a tax. The details of the tax and who specifically is paying for it are not what the argument is concerned about.


14 comments

This is a Weakening question.

This is a very difficult question, to put it mildly. Look at the crazy analytics. But hopefully with a clear-eyed review, we can strip away the difficulties and see that at its core, this is a representative Weakening question where the correct answer choice challenges an assumption. In other words, it gets in the space between the premise and the conclusion.

The historian begins by talking about medieval epistemology. This question is already trying to scare you. Look at how many times this word or its cognate appears in the stimulus. You know the LSAT writers were smirking when they wrote this. But it turns out that it does not even matter what epistemology is. This word could have been anything: theology, biology, etc. The stimulus is just talking about how we determine whether something belongs in a particular set; it just so happens that the set has a scary name, "epistemology."

The historian says because medieval epistemology is a complex subject, intellectual historians have, until recently, failed to produce a definition that would help to determine what should and what should not be included in it. The historian then concludes that, clearly, the solution is to define medieval epistemology simply as “the epistemological beliefs of the medieval epistemologists.”

And why should we believe this? Here comes her premise. That way, if we want to know whether medieval epistemology includes some epistemological claim, we just ask whether any medieval epistemologists believed it. If any did, then it is a part of medieval epistemology; if any medieval epistemologist believed the opposite, then that opposite claim is part of medieval epistemology.

Say you are a medieval epistemologist. I am sure you have a bunch of beliefs about what you ate for lunch, whether your dog is a good dog, etc. But we only care about your beliefs about epistemology, of which you have four. So all four beliefs go into the set of epistemology. Let's examine another medieval epistemologist. This epistemologist has three beliefs about epistemology. So these three beliefs also go in the set. And we just keep doing this.

The historian then says that if one epistemologist had an epistemological belief, X, and if some other epistemologist believed not X, then both X and not X would be in the set. That's the end of the argument.

Immediately, we want to challenge this argument because we recognize that if we do what the historian says, the set could be internally inconsistent. That is, there could be two members (beliefs) that contradict each other in the set. And lo and behold, Answer Choice (D) is right there waiting for us. It says some medieval epistemologists had epistemological beliefs that contradicted the beliefs of other medieval epistemologists. This is exactly our concern above. (D) makes the hypothetical contradiction in the argument actual. So we've weakened the argument!

But have we really though? Nowhere in the argument was it stated that internal consistency was a goal, not by the other intellectual historians nor by our historian who offered the solution. You might be tempted to say that it is a common sense assumption that belief systems must be internally consistent, but no. Because that depends on the purpose of the belief system. If you are building rockets to go to Mars, then your belief system had better be internally consistent or you're going to explode. But these are just historians trying to figure out what counts as medieval epistemology.

Say I wanted to define medieval biology and our historian says to define it simply as the biological beliefs of medieval biologists. So what if some biologists believed frogs were reptiles and some believed frogs were not? This is a contradiction, but what would be the problem here? We are just trying to catalog what people believed. I would expect that in the aggregate of medieval biologists' beliefs, there were at least some internally inconsistent beliefs. Someone had a particular belief and other people disagreed with it, but both are a part of medieval biology. Progress will simply reveal who's right and who's wrong. This is true even today, with modern biologists.

So why would things be different for epistemology? For example, maybe some people thought knowledge is true belief while others thought knowledge is justified true belief. This is fine. Our historian is just trying to figure out what counts as medieval epistemology, and nothing in the conclusion says medieval epistemology has to be internally consistent. So what if internal inconsistency is a feature of medieval epistemology? As uncomfortable as that idea may feel, it is in fact what the historian explicitly says in the last sentence of the argument. Our discomfort arises from an unwillingness to accept that premise because we realize that the consequences of accepting that premise are that we might end up with an inconsistent set. But one more realization will dispel that discomfort: that a consistent set was never a requirement.

Correct Answer Choice (E) says there is much debate as to which medieval thinkers, if any, were epistemologists. This is a much better answer because this answer mined out an assumption the argument made. For our historian’s definition to work, it is presumed that we know who these epistemologists are. Otherwise, how would we know who to even examine out of all the medieval thinkers whose thoughts have survived?

And (E) calls out this assumption by saying it is not clear which thinkers were epistemologists. Now our historian’s solution is not a solution at all because it has just kicked the can down the road. Our original problem was that it was unclear which beliefs should or should not be included in the set of medieval epistemology. If (E) is true, then we still have the same problem because now it is unclear which thinkers should or should not be included in the set of medieval epistemologists.

By the way, it is no accident that (E) came after (D) had trapped a lot of people. But hopefully you see now that (D) latches onto a concern we had that was irrelevant to the evaluation of the argument, whereas (E) latches onto a concern we probably did not think of but, in fact, is highly relevant to the evaluation of the argument.

Answer Choice (A) says medieval epistemologists held some of the same epistemological beliefs as did ancient epistemologists. Great. So they believed the same things as the ancients like Plato. All (A) says is that some thoughts were not original. Maybe this overlap was coincidental, or maybe the plagiarizing medieval epistemologists took things straight from Plato and did not even give him credit.

(A) would have been a bit more relevant if our concern was about original thought or tracing the roots of these thoughts. And note that the historian’s definition still works even if (A) were true. Who cares where these beliefs came from? We are just trying to figure out what belongs in a set.

Answer Choice (B) says the epistemological beliefs of medieval epistemologists depended upon their beliefs about non-epistemological matters. Similar to (A), (B) is mapping out the relationship between epistemological beliefs and the scaffolding they stand upon. For example, in order to have biological beliefs, one first needs to have beliefs about chemistry, physics, and ultimately mathematics. Maybe (B) is hinting that the medieval epistemologists’ epistemological beliefs depended on their beliefs about theology. We are talking about medieval epistemologists, after all. But how does this affect our argument? The solution proposed by the historian still works whether (B) is true or not.

Answer Choice (C) says the writings of most medieval epistemologists include passages that are clearly not about epistemology. Of course they would. I am sure medieval biologists, theologists, or anyone else also wrote about other things. But remember we only care specifically about the epistemological beliefs of the epistemologists. It does not matter that they also wrote about
how the conjunction of Mars and Jupiter made them feel strong, confident, and sexy for the whole month. The historian’s solution ignores these non-epistemological (astrological) beliefs, so even if (C) is true, her proposed solution works as well as it ever did.


75 comments

This is a Flaw/Descriptive Weakening question.

You might be able to figure out the flaw before looking at the answer choices since we're dealing with a repetitive causal logic flaw.

The official says that six months ago, the fines for parking violations on the city’s streets were raised to help pay for the parking garage that had just opened. Since then, parking violations on our streets have dropped by 50 percent. These are our premises, and here comes the conclusion. Hence, if we want there to be even fewer parking violations, the fines should be raised again.

There is an unstated causal assumption. The official makes this recommendation because she assumes that the fines increasing not only coincided with but caused the reduction in parking violations. This could be true, but remember that two things happened: fines were increased and a new parking garage opened. Do we know that it is not the parking garage that caused the reduction? I mean, it's a parking garage. It's for parking cars. I am not saying it has to be, but simply that the official’s reasoning is flawed because she overlooks this. She assumes increased fines —cause→ reduction without considering parking garage —cause→ reduction.

This error can be phrased in a couple of different ways, but let's first look at how it is phrased in the correct answer choice and then play around with alternative phrasings.

Correct Answer Choice (E) says the argument fails to establish that the initial decrease in parking violations was not due to the availability of additional parking spaces (from the newly opened parking garage). This is great. To establish that the argument’s preferred explanation is correct, we have to establish that it is not the alternative explanation that caused the reduction. But like I said, this is not the only way to phrase this flaw. Let’s look at (A).

Answer Choice (A) says the argument takes a possible effect of a reduction to be a possible cause of that reduction. I think one of the reasons why (A) is attractive is that the correct answer could have been phrased similarly to (A). It could have said that the argument takes a possible cause of a reduction to be the definite cause of that reduction. This could have been the correct answer. It would have been using a positive phrasing, "takes a possible cause..." as opposed to the negative phrasing in (E) of “fails to establish...” but that's fine. There are many ways to phrase the error.

But the problem with (A) is that the argument didn't take the effect of the parking violations going down to be the cause of the fines going up. That is, the argument didn't take the loss of revenue from fewer parking violations to be the cause of increased fines.

Answer Choice (B) says the argument takes for granted that raising fines a second time will reduce parking violations at least as much as it did the first time. Takes for granted is also language that the correct answer could have used. The argument "takes for granted that a possible cause of a reduction is the definite cause of that reduction." But, of course, this is not what (B) says. What (B) says is not even descriptively accurate. The official does not say that raising fines will get us another 50 percent reduction. She merely says, “if we want to further reduce,” and does not say by how much.

But (B) is not great even if it passed the descriptive accuracy test. Say we changed (B) and it now reads, “takes for granted that raising fines a second time will reduce parking violations like it did the first time.” This is better since it is a flaw to assume that something will work a second time just because it worked the first time. But (B) still misses the major flaw that it worked the first time.

Answer Choice (C) says the argument fails to take into account the financial benefits the city is now deriving from fines for parking violations. So it did. Okay, (C) is descriptively accurate. But how is this the flaw? Did the official need to talk about these benefits, like how we might use these funds to hire more traffic police or something? This has nothing to do with evaluating the argument.

Answer Choice (D) says the argument takes for granted that people who park their cars illegally would prefer to park their cars legally. Of course people would prefer to park legally. Okay, I know that some people hate being told what to do and believe that it is their right as red-blooded Americans to park on their neighbor's lawn, but the overwhelming majority of people do not.

(D) is indeed an assumption that runs through the whole argument, because whether it is the fines or the new garage that resulted in the reduction, both carry the assumption that people want to park legally. But the best we can do is to say that (D) is descriptively accurate. But it doesn't describe the flaw.


13 comments

This is a Weakening question.

The consumer advocate says some agricultural crops are now being genetically engineered to produce important pharmaceuticals. That's context. However, this development raises the possibility that the drugs will end up in the general food supply, since (premise indicator) if pollen from a drug-producing crop drifts into a nearby field in which an ordinary, non-drug-producing crop of the same species is being grown, the pollen could fertilize that crop and turn it into a drug-producing crop as well.

This is a pretty cookie-cutter structure. We get the context, conclusion, and the premises. The context is that there are some crops, for example, watermelons, being genetically engineered to produce drugs. And the conclusion is that this raises the possibility that these drugs might end up in the general food supply.

Why is this so? Say a farmer is growing regular watermelons for food, and in the next field a pharmaceutical company is growing genetically engineered watermelons for drugs. The consumer advocate is saying that if pollen from this genetically engineered watermelon drifts into our farmer’s field, then it could fertilize the regular watermelons and turn them into drug-producing watermelons. And the farmer might not know this and just ship the watermelons off, which would lead to drugs being in the food supply.

If you do not see the gap in the reasoning already, that's fine. I usually cannot predict the correct answer in Weakening questions unless they are really obvious, so I am going to use process of elimination.

Answer Choice (A) says that as far as scientists know, none of the pharmaceuticals produced by genetically engineered crops would present any danger to public health if they were present in the general food supply. If you picked (A), the LSAT writers might have successfully distracted you from your objective.

(A) is saying that even if the conclusion was true, there would not be a public health danger. Whew, crisis averted! This is not how you weaken an argument. To weaken an argument, you have to say that even if the premises are true, the conclusion might not be supported, not that even if the conclusion were true, there wouldn't be a danger to public health. We are not trying to accept the conclusion and convince the consumer advocate that this is not a big deal.

Answer Choice (C) is similar. (C) says the genetically engineered crops that produce pharmaceuticals are not among the crop species that comprise the largest portions of the general food supply. What does this even mean? First, imagine the set of general food supply. Next, what "crop species comprise the largest portions" of that set? Probably the staple crops like corn, wheat, potatoes, etc. (C) is saying that the genetically engineered crops are not in this set of staple crops. Oh, that's good. I mean, we already used watermelons for illustration so it already wasn't in the set. But wait, does that matter?

Like (A), (C) is trying to make you accept the conclusion and convince you that it is not a big deal. It is fine because genetically engineered crops only comprise a small portion of the general food supply. I concede that both (A) and (C) might be latching on to and attempting to mitigate the concern which motivated the consumer advocate in the first place, but neither is weakening her argument.

Answer Choice (B) says if pollen from a genetically engineered crop is prevented from drifting into fields in which ordinary crops of the same species are being grown, then there is no risk of the pollen fertilizing the latter crops. Sure, but the consumer advocate is saying that if the pollen drifts, then the resulting crop will be a drug-producing crop. (B) just says if pollen does not drift, then there is no danger. That doesn't work. If the premise places us in a hypothetical world where pollen drifts, refusing to be in that world won't do anything to weaken the argument.

Answer Choice (E) says that if pollen from a drug-producing crop turned an ordinary crop of the same species into a drug-producing crop, it would be possible for scientists to identify the latter crop. In order for (E) to weaken, you need to make a pretty big assumption about the consequences of the scientists' capacity to identify the latter (drug-producing) crop. What will they do? Will they act to stop the accidental crop from entering the general food supply? Will they throw up their hands and say, "Oh well"? (E) is silent.

Also notice that (E) does not tell you anything about when the scientists are able to identify this. It just says it is possible. But surely timing matters. Imagine a scientist is enjoying her watermelon and notices that it tastes kind of funny, like the watermelon she produced in a lab. She now knows something bad happened, but drugs are now already in the food supply.

Correct Answer Choice (D) says that in crops genetically engineered to produce pharmaceuticals, the drugs are not present in any part of the plant used for food in ordinary crops of the same species. This is a classically correct weakening answer choice. It reveals a naive assumption we made, which was that it is the flesh of the watermelon that would contain the drugs.

(D) says this is not the case. (D) says that it is the leaves or the stems of the watermelons that contain the drugs. If (D) were true, then even if food-producing watermelon crop turned into drug-producing watermelon crop, the drugs would not end up in the general food supply.


6 comments

This is a Fill in the Blank question.

The question stem says “the conclusion of the argument follows logically if,” which should remind you of Sufficient Assumption questions. And sure enough, there is a premise indicator “since” right before the blank. We are looking for a premise to fill in this blank such that the conclusion follows logically. Once you figured this out, hopefully you will soon see that the rest of this question is just a pure exercise in formal logic. In fact, you drilled this specific argument form in the core curriculum. This is not to say that this is an easy question, only that there is a fair amount of predictability in Logical Reasoning.

The stimulus says that most of the members of Bargaining Unit Number 17 of the government employees’ union are computer programmers. I am going to refer to this as 17. And the relationship between 17 and computer programmers is not that of complete subsumption, but a most relationship.

17 —m→ computer programmers

This is the only premise, and the question jumps straight to the conclusion. Thus it is certain that some of the government employees who work in the Hanson Building are computer programmers. In other words, there must be at least one overlap between these groups.

Hanson ←s→ computer programmers

So the conclusion is a some relationship, but the premise gives us a most relationship. If you recall from the core curriculum, you can use a most relationship to get to a some relationship by supplying another most relationship or something stronger. Unfortunately, all of the answer choices are most statements. So while you can use the process of elimination, it is faster to recall how you can build the correct premise to arrive at the conclusion we want.

The way to do that is to supply this missing premise:

17 —m→ Hanson
17 —m→ computer programmers
_______________
Hanson ←s→ computer programmers

How does this argument work? Say there are three members in 17, and most of them work in the Hanson building. This means I have to take at least two members and scoop them into Hanson. But I also have to take most of 17, at least two members, and scoop them into computer programmers as well. Then these two scoops must intersect by at least one member. And this is exactly what Correct Answer Choice (D) gives us. It says most of the members of Bargaining Unit Number 17 work in the Hanson Building.

Answer Choice (C) says most of the government employees who work in the Hanson Building are members of Bargaining Unit Number 17 (Hanson —m→ 17), so we can chain this to our premise to get:

Hanson —m→ 17 —m→ computer programmers

But two most statements chained up like this don't yield the desired conclusion. Say we took most members of Hanson and dropped them into 17. But 17 could be a humongous set and Hanson could be a tiny set, so this is just a drop in the bucket. Now if we scoop most of 17, there is no guarantee that we will capture a member of the Hanson set. We could, but we could just as well not.

If this does not make sense, you can either review the core curriculum or, better yet, try assigning numbers. Assign a small number for Hanson and a large number for 17 and see how you can make the two most relationships true while making the conclusion that Hanson ←s→ computer programmers false.

Answer Choice (B) is kind of interesting. It says most members of the executive committee of Bargaining Unit Number 17 work in the Hanson Building. Who are these executive committees? We can infer that the executive committee of 17 is some subset of 17. But this subset (the executive committee) could be very small. Nothing in (B) tells us anything about the size of the executive committee. If it's very small, then the fact that most of them are in Hanson merely amounts to a claim that some (not most) members of 17 work in Hanson. That's not powerful enough. Contrast this with the much more powerful most relationship in (D), where we did not carve down into the subset first before supplying the most relationship. (D) just outright said that most of 17 belongs to Hanson. (B) says first, carve down into a subset of 17. Most of that subset belongs to Hanson.

Answer Choice (A) says most of the government employees who belong to Bargaining Unit Number 17 but are not computer programmers work in the Hanson Building. This is really bad. (A) is looking at the intersection of 17 and not computer programmers. (A) takes a most cut of that set and puts them into Hanson. But that's useless because this intersection is already a set of people who are not computer programmers. How does that help us establish that some people in Hanson are computer programmers?

Answer Choice (E) says most of the people who work in the Hanson Building are government employees. So who are the government employees? Well, they are a superset of 17. Remember, 17 stands for Bargaining Unit Number 17 of the government employees' union. So everyone in 17 is a government employee, but government employee is a superset of 17.

(E) says to take the people in Hanson and drop most of them into the government employee superset. Okay, but where do they land in this large government employee superset? We don't know. Will they even land in the 17 subset? No idea.


17 comments

This is a Parallel Flaw Method of Reasoning question.

The stimulus says that every Labrador retriever in my neighborhood is a well-behaved dog. Here we can kick the idea of dogs and my neighborhood up into the domain and say lab → well behaved. However, no pet would be well behaved if it were not trained. Dogs are a subset of pets, so you can kick the idea of pets into the superset and say /trained → /well behaved. The contrapositive is well behaved → trained. This allows us to hook up our conditions to get the following:

lab → well behaved → trained

We can connect the second claim about pets to a claim about dogs because if a rule pertains to all pets, then it must also pertain to all dogs since dogs are a subset of pets.

And here comes the conclusion. Thus it is training, not genetic makeup of the breed, that accounts for these Labrador retrievers’ good behavior. This is a causal claim with a positive and a negative component. The positive component is that training accounts for good behavior, and the negative component is that it is not the genetic makeup of the breed that accounts for good behavior. Note that “genetic makeup” refers to the fact that the dogs in question are genetically of the Labrador breed. So I would write:

lab → well behaved → trained
________________________________________________
training —cause→ well behaved
not (lab* —cause→ well behaved)

The asterisk is to indicate that it is not exactly the fact that the dog is a Labrador, but a variant of this idea (genetic makeup of the dog) that does not cause good behavior.

This is a strange argument. The premise establishes that training is a requirement for being well behaved so it does seem like there's some causal power flowing from training to behavior. But how can we be sure that genetics has nothing to do with it? There's no support for that claim at all.

While it's good practice to think about why the argument is weak, this is a Parallel Flaw question so you don't really have to in order to get the question right. You can just do a formal mapping, like this:

▢ → ◯ → △
________________________________________________
△ —cause→ ◯
not (* —cause→ ◯)

Correct Answer Choice (B) gives us this. Whenever it snows there are relatively more car crashes on the highways. Kick highways up into the domain. Within this domain, snow → more crashes.

Then (B) says yet in general, there would not be car crashes unless people were careless. Here, “in general” is the superset of the highways, and “unless” is group 3 negate sufficient, which means carelessness is a necessary condition for crashes (crashes → carelessness). All crashes, of course, includes the “more crashes” on snowy days, so:

snow → more crashes → carelessness

And (B) gives us the correct causal conclusion with the positive and the negative components. (B) says it is not icy roads, but carelessness, that causes car crashes when it snows. Notice how the icy roads are not exactly snow, but a version of it, similar to what we had for Labradors and genetic makeup.

carelessness —cause→ crashes
not (snow* —cause→ crashes)

Answer Choice (A) says all the students at Bryker School excel in their studies. Kick students up into the domain. The stimulus was talking about a subset of dogs, the Labrador retrievers. Here, we are talking about a subset of students, the ones at Bryker. So within this domain of students, Bryker → excel. However, students at Bryker School would not excel if they did not have good teachers (/good teachers → /excel). Notice this claim is not a more general claim and therefore already a bit off. But let's keep going and take the contrapositive, excel → good teachers, and chain it up:

Bryker → excel → good teachers

(A) lacks the domain shift we had with the pets, but I suppose this does not preclude it entirely. (A) is so far just a bit worse than (B), but what happens next really kills (A). (A) should have said:

good teachers —cause→ excel
not (*Bryker —cause→ excel)

In other words, it is the good teachers that produce (or some other causal verb) excellence, not the Bryker school or some asterisk version of it. But (A) instead says all schools should hire good teachers if they want their students to excel. This is a prescription. It's a policy recommendation. It's not a causal explanation. (A) maybe presumes a causal relationship, but then on the basis of that, makes a recommendation. In addition, notice that the conclusion in the stimulus is limited to the Labrador retrievers in the neighborhood. It doesn't attempt to draw a more generalized conclusion from the premises. Yet (A) does. (A) tries to draw a more generalized conclusion about all schools based on premises about the Bryker School.

Answer Choice (C) says every musician I know is a good dancer (musician → good dancer) and every mathematician I know is a bad dancer (mathematician → bad dancer). This already does not look good, but let’s keep going. There's an implicit premise that good dancers are not bad dancers. So, if musicians are good dancers, then they are not bad dancers, then they are not mathematicians.

musician → good dancer (→ /bad dancer) → /mathematician

Okay, so this at least kind of resembles our stimulus. What we need for (C) to work is to find this in the conclusion:

/mathematician —cause→ good dancer
not (*musician —cause→ good dancer)

In other words, not being a mathematician causes one to be a good dancer and being a musician* (like having a good sense of rhythm) doesn't. However, (C) gives us the opposite. It says it is a sense of rhythm, not the ability to count (mathematician*), that is responsible for good dancing.

Answer Choice (D) says all of the good cooks in my country use butter, not margarine, in their cooking. Let's kick “my country” into the domain. And within my country, good cook → butter and good cook → /margarine. Already, this is not going to work. You are not going to be able to hook up these two conditions by taking the contrapositive. (D) then concludes that if you want to be a good cook, you must use butter, not margarine.

We already know that (D) isn't right. But is (D) a good argument? Nope. It's terrible. Do good cooks in other countries also use butter and not margarine? And is using butter and not margarine what makes cooks good? The conclusion is a causal claim while the premise is merely an observational claim. I am sure that not too long ago, you could have made an observational claim that all of the best chefs in the world are men, and therefore, what... if you want to be a top chef, you must be a man?

Answer Choice (E) says all of the students in my algebra class received an A, even though none of them can solve word problems. The domain here is the algebra class, and within this domain we have student → A and student → /solve word problems. Like (D), we cannot hook up these two conditions by taking a contrapositive. But (E) has an additional premise. It says that no student who is unable to solve word problems has an adequate understanding of algebra (/word problems → /understanding). This is actually not bad because (E) now makes a general claim that applies beyond the algebra class and this claim chains up:

student → A
student → /word problems → /understanding

The second conditional chain resembles our stimulus, so we can ignore student → A. Now, what I need for (E) to work is the following causal conclusions:

/understanding —cause→ /word problems
not (*student —cause→ /word problems)

In English, it is the lack of understanding of algebra that's responsible (cause) for the inability to solve word problems, not the fact that one is a student*. The conclusion in (E), however, is that the students in my class received A's not because they did any good work. Where did “good work” even come from? And also, remember we are ignoring student → A since the parallel of this is not present in the stimulus.


10 comments

This is a PSA question.

The stimulus says the builders of the Glen Veil apartment complex will not complete the complex unless a road connecting it to the town of Sierra is built. “Unless” is group 3 which means negate sufficient, so for the builders to complete the apartment complex, it is necessary that the connecting road be built (complete complex → road built). This is the first claim, and it is a conditional claim.

The next claim is a causation claim, not a conditional claim. It says the completed apartment complex would strengthen Sierra’s economy, and a stronger economy would benefit every Sierra resident. So I would write:

complete complex —cause→ strong economy —cause→ every resident benefits

And the prescriptive conclusion is that the residents of Sierra should vote to fund the construction of the proposed road.

Before we get into the answer choices, can you state in very general terms the descriptive-premise (P) → prescriptive-conclusion (C) we are looking for? It is something like if a construction project is necessary to another construction project whose consequence is a strengthened economy or some benefit to everyone, then residents should vote in favor of the first construction project. Obviously, it does not have to be this exact wording, but you are looking for some variant of this P → C. But watch out for C → P.

Which is the recurring trap that shows up in Answer Choice (B). It says the residents of a town should not vote in favor of a local tax to fund the construction project unless that construction project will produce results that benefit all of those residents. “Unless” is negate sufficient. If residents should vote to fund the construction project, then every resident benefits. You can bet money all day long that they will give you the right content but in the wrong logical order in PSA questions.

(B) should say if the construction project will produce results that benefit all residents, then the residents of the town should vote for it. The construction of the road is necessary for the construction of the apartment which benefits every resident, so you can say that the construction of the road will also produce results, albeit indirectly. This is not logically airtight, but you will see later on in (E) why this could work in the context of a PSA question.

Answer Choice (A) is also wrong, but for a different reason. (A) says if a construction project is necessary in order to strengthen a town’s economy, then the residents of the town should vote in favor of a local tax to fund that construction project.

While (A) is capable of delivering the desired conclusion, the premise does not trigger (A)'s sufficient conditions. We were never told of any necessary conditions to strengthen a town's economy. Building the road is necessary for the apartment complex which in turn would strengthen the economy, but that doesn't mean building the road is necessary for strengthening the economy. There may well be plenty of other ways to strengthen the economy without having to build that road.

One way you could fix (A) is by fixing the stimulus. The stimulus could say that in order for our economy to be strengthened, it is required that the apartment complex be completed. You could also fix (A) itself. If a construction project is necessary for another construction project whose consequence is that of a strengthened economy, then the residents should vote to fund it. This would trigger our stimulus.

See if you can come up with an analogy. Here's one. For me, wearing running shoes is necessary in order to go for a long run. Going for a long run results in me feeling better. (A) wants to claim that wearing running shoes is necessary for me to feel better.

Correct Answer Choice (E) says anyone who would benefit from the results of a construction project should vote in favor of a local tax to fund that project. First of all, (E) has the premise and the conclusion in the right location. If you would benefit, then you should vote in favor. This is the P → C.

You might object to (E) by pointing out that the construction project referred to in (E) could be the construction of the apartment complex. Yes, but there is nothing in (E) that forces only this interpretation. We could have just correctly identified the construction project as the building of the road.

You might also point out that building a road is only a necessary condition for the apartment complex, and the benefits to every resident are consequences of the apartment complex, not the road. You might say that we do not know the benefits of the road since it is just a necessary condition.

But this is not a Sufficient Assumption question. This is a PSA question. In context, the result of building the road cannot be nothing. The stimulus said the apartment complex will not be completed unless the road is built, so it seems reasonable to say that the result of the road being built is that the apartment gets completed, which has its attendant benefits, some of which can be attributed to the road.

I admit this is not logically airtight, but we are in PSA territory and we're dealing with causal logic. It is much more reasonable to make this argument as opposed to one required for (A), where we would have to say that just because the apartment complex would result in a strengthened economy, there is no other way to strengthen the economy. Meanwhile, (E) is only asking us to say that the consequences of the apartment complex can accrue to the road as well, given the relationship between the two projects.

Answer Choice (C) says whenever a town funds a construction project, it should do so by means of a tax rather than a debt. Nobody is contemplating a debt. If I was trying to argue that a town should fund its projects with tax and not with other sources, then maybe (C) would contribute. But as it stands, (C) is totally useless.

Answer Choice (D) says only those residents who will benefit from the results of a construction project should be required to pay a tax. The rule here sets us up for contraposing in order to yield the result that certain residents shouldn't be required to pay a tax. That's not what we want. We are only trying to argue that residents should vote in favor of a tax. The details of the tax and who specifically is paying for it are not what the argument is concerned about.


14 comments

This is a Strengthening question.

Be careful not to mistake the question stem for a Most Strongly Supported question. In a Strengthening question, you are choosing an answer choice that adds to the support of the argument, while in an MSS question, you are taking the stimulus to be true and finding the answer choice that receives the most support.

Hopefully we got over that hurdle, but there are many more hurdles in the argument itself. You first have to understand how the “even if” is functioning in the stimulus as well as be able to take the contrapositive of the premise. However, this is not just an exercise in formal logic like an SA or a PSA question where you just map out the conditionals. In fact, this is actually more of a phenomenon-hypothesis question where the correct answer choice blocks an alternative hypothesis.

Let’s look at the stimulus. It says you should only buy a frying pan that has a manufacturer's warranty. That sounds like a command. And as if expecting an objection, the author says even if it requires paying a little bit more, I should still go for the warranty. But what if I am not the kind of person that would ever find the warranty form, fill it out, and mail it in? And as if the author had also anticipated this second objection, we are told that even if you would never bother seeking reimbursement should the pan not work, you still need to buy the pan with the warranty.

Do you see what the “even if” is doing? Imagine if the author had said that one should buy a frying pan that has a manufacturer's warranty, full stop. That full stop still completely anticipates any potential objections. So in a sense, the “even if” is not doing much. It is just reinforcing that you do not get out of this rule with these possible objections. If you are buying a frying pan, you'd better buy one with a manufacturer's warranty (buy pan → warranty).

And the premise to this conclusion is that manufacturers will not offer a warranty on a product if doing so means that they will need to reimburse many customers because the product did not work well or last long (reimburse many → no warranty). Let’s take the contrapositive of this. If a manufacturer does offer the warranty, then it is not the case that they need to reimburse many customers. (warranty → not reimburse many). And this is why you should only buy things with manufacturer's warranties. So the unstated assumption in this argument is that a manufacturer's warranty is an indicator of high quality, which means that not having to reimburse many customers is an indicator of high quality.

That is the unstated assumption, but let’s evaluate this assumption. Is it actually the case that not having to reimburse many customers is an indicator of high quality? This is where your phenomenon-hypothesis framework comes in. Let’s say All-Clad, an American manufacturer of frying pans, sells many frying pans but very rarely reimburses customers because of bad products. How do you explain this phenomenon?

One hypothesis might be that they make good products. It works well and lasts long, so not many customers seek reimbursement. And this is what the author presumes. But can you think of an alternative hypothesis? Maybe seeking reimbursement is a difficult process. You have to fill in a thousand different forms and call five different departments. Or maybe the particular market segment for frying pans is just too lazy, like me, to seek reimbursement. Both hypotheses would weaken the argument because now the fact that not very many people call in the warranties is no reflection on product quality.

Correct Answer Choice (A) says that most people who buy a frying pan with a manufacturer's warranty would seek reimbursement should the pan fail to work or last long. This wrecks the hypothesis above. (A) strengthens the argument by knocking out a couple of these competing hypotheses so that the product being high quality is more likely to be the hypothesis that explains the low reimbursements. (A) has identified precisely where the argument is vulnerable and patched it up. It has reaffirmed an assumption the argument already made.

Answer Choice (B) says that all of the frying pans currently on the market covered by warranty work at least as well at the time of purchase as any of the pans not covered by warranty. On first blush, you might think (B) strongly supports the conclusion because it seems to provide another reason to buy the warranty pans. But this is a pretty common trap the LSAT writers use in Strengthening questions, which is to come up with independent reasons to support the conclusion. Unlike (A), (B) does not address any of the premises or unstated assumptions and just independently supports the conclusion.

But if you examine (B) closely, you will see it does not even independently provide you with a reason to buy the pans with warranties. In order for (B) to independently support the conclusion, we need to assume that it is really bad if the non-warranty pans worked better than the warranty pans at the moment of purchase. If we make that assumption, then (B) helps by precluding that possibility. But we don't need to make that assumption. Would it actually be really bad if the non-warranty pans worked better than warranty pans at the moment of purchase? Frying pans are not disposable like plastic forks. Durability matters. So what I care about is whether the non-warranty pan can keep up its quality, because I know if the quality starts degrading on my warranty pan, I can return it. So maybe I care a little bit that non-warranty pans are better than the warranty pans at the time of purchase, but it is not nearly as important as it might initially have seemed.

(B) is analogous to trap answer choices in Weakening questions that look like they are contradicting the conclusion, but upon closer inspection, they do not. This is a Strengthening question so (B)'s trap is to give you something that looks like it is supporting the conclusion, but upon closer inspection, it does not.

Answer Choice (E) is similar to (B). (E) says most frying pans’ manufacturer’s warranties provide for full customer satisfaction. So (E) gives us yet another reason to buy a pan with a warranty. (E) might be a little bit better than (B) because it supports the conclusion more strongly, but (E) still shares the same flaw as (B), which is that it completely ignores the argument. (E) is not entirely irrelevant, just as (B) was not entirely irrelevant in that how good the pan is at the time of purchase surely matters some. But neither (E) nor (B) meaningfully engages with the argument.

Answer Choice (D) says the most expensive frying pans are the ones most likely to work well for many years. But this information is not helpful because we do not know if the most expensive frying pans have a warranty or not. The author only says that one should go for the warranty even if it requires paying more, and never says warranty pans actually do cost more or less.

Answer Choice (C) says the more a frying pan costs, the more likely it is to be covered by a manufacturer's warranty. I suppose (C) is better than (D) because it is at least correlating cost to warranty. But it still does not do much since the author already said to forget the costs. The author thinks a manufacturer's warranty is an indication of the product's quality, so harping on about costs is not going to affect the argument.


38 comments

This is a Weakening question.

The journalist says when judges do not maintain strict control over their courtrooms, lawyers often try to influence jury verdicts by using inflammatory language and by badgering witnesses. And these obstructive behaviors hinder the jury's efforts to reach a correct verdict. Therefore, whenever lawyers engage in such behavior, it is reasonable to doubt whether the verdict is correct.

Does the conclusion follow? We know obstructive behaviors hinder the jury's effort to reach a correct verdict. But hinder isn't the same thing as prevent. That assumes the hindrance is effective. It's possible that the hindrance prevents the jury from reaching a correct verdict but it's also possible that the hindrance is simply an annoyance, something that the jury can shrug off. The point is that to hinder is just that, to hinder. It's not as dispositive as the argument wants us to believe.

Answer Choice (A) says court proceedings overseen by judges who are strict in controlling lawyers' behaviors are known to result sometimes in incorrect verdicts. I am already not interested in (A) because the argument is contemplating a specific situation where judges do not maintain control over their courtrooms. (A) does not engage with this premise and tells us that in a different world where judges do control their courtrooms, juries also reach wrong verdicts.

To make (A) better, we can edit the conclusion to say that only when lawyers engage in such behavior is it reasonable to doubt the veracity of the verdict. This version of the conclusion is saying that when we have a reason to doubt a verdict, it must be because of obstructive behavior. And (A) would weaken by illustrating a counterexample: juries mess up even when lawyers do not engage in obstructive behavior.

You will see a very similar cookie-cutter wrong Answer Choice (B) in Question 24 of this section, which also tells you to forget the situation at hand and talks about a different world. Just as you find patterns that repeat in the stimulus and correct answers, you will find recurring patterns in the wrong answer choices as well.

Answer Choice (B) says lawyers tend to be less concerned than are judges about whether the outcomes of jury trials are just or not. This is a comparative claim and somewhat obvious. We have an adversarial system where lawyers are concerned about winning and judges are concerned about justice. At best, (B) explains why lawyers have to be kept in check because they are not so concerned about justice.

Answer Choice (C) says people who are influenced by inflammatory language are very unlikely to admit they were influenced by such language. Let’s give (C) a charitable reading and say that the people influenced by inflammatory language here are the jurors. But even with this reading, (C) does not weaken the argument. If jurors were influenced, it does not matter if they are humble or reflective enough to admit that they were. If anything, (C) just tells us that this hindrance might actually lead to reasonable doubt about the verdict.

Answer Choice (E) says the selection of jurors is based in part on an assessment of the likelihood that they are free from bias. This is true in reality. For example, in a corporation versus an individual case, you would want jurors who are not already biased against corporations. However, the argument is not about bias. (E) would have been better if it said the selection of jurors is based in part on an assessment of the likelihood that they are free from being influenced by inflammatory language.

But even then, (E) would not be a great answer choice because it is denying the premise that obstructive behaviors do hinder the jury's effort to some extent. And while you do sometimes see explicit contradictions to a premise in the LSAT, this is generally not how the test writers weaken arguments.

Correct Answer Choice (D) says obstructive courtroom behavior by a lawyer is seldom effective in cases where jurors are also presented with legitimate evidence. This is the only answer choice that addresses the gap between the premise and the conclusion.

Remember we said that just because obstructive behaviors hinder the jury's efforts, we cannot assume that this hindrance is effective. And (D) confirms this. It says there's this other factor, legitimate evidence, that renders obstructive behaviors ineffective.

You might object and say (D) requires an assumption that cases where jurors are presented with legitimate evidence are most cases, or at minimum, the cases where lawyers engage in obstructive behaviors. But this is not an unreasonable assumption. We have a functioning judicial system with robust rules of evidence, and in the vast majority of cases, there is legitimate evidence. You might think it is unfair to be expected to know this, but what you are expected to know is how to identify the assumption that (D) requires and compare it to assumptions required by the other answer choices. While (D) may not be the perfect answer choice, it still is the best answer choice.


5 comments

This is a PSA question.

Mateo says global warming has caused permafrost to melt under several arctic villages, forcing all of their inhabitants to relocate at great expense. Then he says that pollution from automobiles is a major contributor to global warming (another premise). Now he's given us a causal chain starting with automobiles and leading to pollution, then global warming, then melting permafrost, and finally, expensive relocation. Here comes the conclusion: the automotive industry should be required to help pay for the villagers' relocation. We have descriptive causal premises leading to a prescriptive conclusion.

In PSA questions, we have the premise (P) and the conclusion (C) and are generally looking for a P → C "rule." And the reason the question stem includes the word "principle" is that the P → C could be stated in terms more general than what you see in the answer choices. And there is a large degree of freedom in how general they will be.

For example, the correct answer could generalize a lot and say any "entity" whose "actions" have some negative consequence must pay for the cost of that consequence. You can take this general principle and apply it to this argument, but you can also apply it to a bunch of other arguments. The correct answer could generalize less and say if an "industrial product" has some negative downstream consequences, then the "manufacturer" must pay to cover the costs of those consequences. This one would apply to a smaller set of arguments than the more generalized principle of entities.

Most PSA questions lay out recurring traps. We find one such trap, that of starting the bridge at the conclusion in Answer Choice (E). It says an industry that contributes to global warming should be required to pay for resulting damage to specific communities only if it has a general obligation to help pay for all damage produced by global warming.

An industry being required to pay is the desired conclusion. So that concept cannot be in the sufficient condition. At best, this rule allows the stimulus to fail the necessary condition (say we had a premise that amounted to the auto industry not having a general obligation to help pay for all damage), therefore concluding the failure of the sufficient (auto industry should not be required to pay for damage to specific communities). That's not what we want. This answer choice is logically wrong.

(E) would have been correct if it said an industry that contributes to global warming should be required to help pay for resulting damage to specific communities, full stop.

Correct Answer Choice (D) says any industry manufacturing a product whose use contributes to costly damage for others should be liable for damages generated by that product's use. This is the P → C rule. The information in our premise triggers the sufficient conditions: any industry (automobile) manufacturing a product (cars) whose use contributes to costly damage (melting-permafrost-induced relocation) for others (arctic villagers). Hence, we can draw the necessary condition as the conclusion: the automotive industry should be liable for damages.

Answer Choice (A) says any industry has an obligation to pay for any damage that it should have known would result from its activities. (A) is pretty good if you strike out “it should have known.” But as it stands, that condition doesn't match the premise and so the sufficient condition in (A) does not trigger. While it might be common sense that cars generate pollution, the long causal chain in the stimulus makes it less reasonable to say that the automotive industry should have known about the results of their activities on arctic villages. (A) exhibits the recurring defect of starting the bridge at the wrong location.

Answer Choice (B) says manufacturers should be required to produce goods in a way that minimizes harm to people and the environment. (B) just tells the automotive industry to change the way it produces cars, but does not help the villagers get paid. This rule delivers the wrong results. Also a recurring defect.

Answer Choice (C) says when the use of a product causes damages, governments should not be required to pay for the damage unless those responsible for manufacturing the product are also required to pay. (C) is saying that if the government is required to pay, then it must be that manufacturing is required to pay. Or, contrapositive, if the manufacturer isn't required to pay, then neither is the government.

This is a strange principle to talk about when you are trying to get arctic villagers paid. Maybe (C) thinks Mateo’s next argument is to petition the government to compensate these villagers, and it is saying that if you want the government to pay, you must first establish that manufacturing has to pay. How bizarre.

This rule also exhibits the recurring defects of failing to match the premise and failing to match the conclusion.


2 comments